Taylor Series and Random Variables

Click For Summary
Using a Taylor Expansion around the mean of a variable allows for the approximation of the mean and variance of a function. In the case of the function f(y) = e^(y), taking the expectation reveals that the bias correction factor in Equation A overcorrects when the residual series has negative skewness. The skewness, defined as p = E((y-u)^3)/(o^3), indicates how the distribution deviates from symmetry. The final term in the expansion, which involves the expectation of the cubic, suggests that the negative skewness leads to an underestimation of the mean. This highlights the importance of considering skewness when applying Taylor expansions for approximations.
GottaLoveMath
Messages
3
Reaction score
0

Homework Statement



A standard procedure for finding an approximate mean and variance of a function of a variable is to use a Taylor Expansion for the function about the mean of the variable. Suppose the variable is y, and that its mean and standard deviation are "u" and "o".

f(y) = f(u) + f'(u)(y-u) + f''(u)(((y-u)^2)/2!)) + f'''(u)((y-u)^3)/3!)) + ...

Consider the case of f(.) as e^(.). By taking the expectation of both sides of this equation, explain why the bias correction factor given in Equation A is an overcorrection if the residual series has a negative skewness, where skewness p of a random variable y is defined by

p = E((y-u)^3)/(o^3)

Equation A = x^hat_t = e^(m_t + s_t)*e^((1/2)(o^2))

where x_t is observed series, m_t is the trend, s_t is seasonal effect

Homework Equations

The Attempt at a Solution



Im not even really sure where to start. If someone could point me in the right direction, it would be greatly appreciated
 
Physics news on Phys.org
Try what is suggested. If you begin with expected values like this:
<br /> E(f(y)) = f(\mu) + f&#039;(\mu)E(y-\mu) + \frac{f&#039;(\mu)}{2!} E((y-\mu)^2) + \frac{f&#039;&#039;&#039;(\mu)}{3!} E((y-\mu)^3)<br />

What do the assumptions tell you about the final term (term with the expectation of the cubic)?
 
Question: A clock's minute hand has length 4 and its hour hand has length 3. What is the distance between the tips at the moment when it is increasing most rapidly?(Putnam Exam Question) Answer: Making assumption that both the hands moves at constant angular velocities, the answer is ## \sqrt{7} .## But don't you think this assumption is somewhat doubtful and wrong?

Similar threads

Replies
5
Views
2K
  • · Replies 2 ·
Replies
2
Views
2K
  • · Replies 2 ·
Replies
2
Views
1K
  • · Replies 4 ·
Replies
4
Views
2K
  • · Replies 27 ·
Replies
27
Views
2K
Replies
6
Views
2K
Replies
2
Views
2K
Replies
6
Views
3K
  • · Replies 6 ·
Replies
6
Views
2K
  • · Replies 6 ·
Replies
6
Views
2K